开发者:上海品职教育科技有限公司 隐私政策详情

应用版本:4.2.11(IOS)|3.2.5(安卓)APP下载

Suechen · 2020年07月11日

问一道题:NO.PZ2018070201000102 [ CFA I ]

问题如下:

Given the market risk premium is 7% and risk free rate is 4%, please calculate the expected return for security 1 based on CAPM.

选项:

A.

12.0%.

B.

15.2%.

C.

17.5%.

解释:

B is correct.

Based on the CAPM, E(Ri) = Rf + βi[E(Rm)–Rf]

So the expected return of Security 1 equals to

Security 1’s expected return= 4%+1.6*(7%)=15.2%.

答案错了吧,为什么不是4%+1·6*(7%-4%)
2 个答案

Kiko_品职助教 · 2024年07月23日

嗨,努力学习的PZer你好:


E(Rm)是市场组合的收益率,而不是市场风险溢价。

----------------------------------------------
加油吧,让我们一起遇见更好的自己!

丹丹_品职答疑助手 · 2020年07月13日

嗨,从没放弃的小努力你好:


同学你好,题干中给的是market risk premium=[E(Rm)–Rf]


-------------------------------
加油吧,让我们一起遇见更好的自己!


vvnsw · 2024年07月23日

E(Rm)不是market risk premium?

  • 2

    回答
  • 0

    关注
  • 420

    浏览
相关问题